In dynamics, do we assume springs always resist motion?

In summary: And most importantly, this set-up won't affect the stiffness matrix that would be formed for the system pictured? Which, before this confusion, I was certain should be:[2k, -k-k, 2k]
  • #1
Leo_m87
5
0
I'm making a vibration rig that represents a classic 2 mass, 3 spring system (See attached Image).
(To clarify the image - there will be no forcing frequency. m=m1=m2 and k=k1=k2=k3)

I'm using extension springs in the vibration rig. I plan to displace them in extension at an equilibrium point that will have them remain in tension throughout harmonic motion. The springs will therefore always be in tension and will always only exert a tension spring force on the mass. My question is:

In the equations of motion do we always assume the spring constant will exert force in both tension and compression? In other words, If compressed it pushes away and if stretched it pulls back. If so, how will this affect the system I've proposed above? A system where closing the spring it is still in tension and not opposing the motion.

Thanks for taking the time to read this. Any help is appreciated.
 

Attachments

  • new doc 20_3.jpg
    new doc 20_3.jpg
    30 KB · Views: 394
Last edited:
Physics news on Phys.org
  • #2
Hello Leo, :welcome:

You should distinguish between real springs (with thick wires, so they have a minimum length. And with a finite length of the wire they are made up from) and 'ideal physics springs'. The latter theoretical springs do not oppose motion. Instead they oppose displacement, as is is well shown by the spring equation ##F = -k\; x##.

Sometimes - such as for your rig - you want to optimize the situation to ensure this ##F = -k\; x## behaviour occurs in a range of x (positive and negative) that is as wide as is reasonable. Meaning you want to pre-stretch real springs somewhat.
 
  • #3
It will just raise the force or lower the force. Lowering the force is the same as applying an additional force in the opposite direction. So, in short, you don't need both tension and compression of the spring to accomplish the same action.
 
  • #4
Chestermiller said:
It will just raise the force or lower the force. Lowering the force is the same as applying an additional force in the opposite direction. So, in short, you don't need both tension and compression of the spring to accomplish the same action.

Thanks for your reply! This was my intuitive sense of what would happen, but I managed to get confused anyway. Can you just confirm a little further...

If each spring was initially stretched by 10 mm (Apologies to anyone not using metric) there would be spring forces of +k(10) and -k(10) acting on each mass (equilibrium). Then, if the mass is displaced further to the right (+ve direction) by another 5 mm the forces now become +k(15) and -k(5)... Your point is that this has the same net effect as if the compressed spring pushed back ... in other words the other spring being in greater tension is producing the equivalent force?

And most importantly, this set-up won't affect the stiffness matrix that would be formed for the system pictured? Which, before this confusion, I was certain should be:
[2k, -k
-k, 2k]
 
  • #5
BvU said:
Hello Leo, :welcome:

You should distinguish between real springs (with thick wires, so they have a minimum length. And with a finite length of the wire they are made up from) and 'ideal physics springs'. The latter theoretical springs do not oppose motion. Instead they oppose displacement, as is is well shown by the spring equation ##F = -k\; x##.

Sometimes - such as for your rig - you want to optimize the situation to ensure this ##F = -k\; x## behaviour occurs in a range of x (positive and negative) that is as wide as is reasonable. Meaning you want to pre-stretch real springs somewhat.
Thanks for your reply, and for the welcome!

I had a feeling that my use of terminology wouldn't be specific enough! It's easy to be too ambiguous, or use the wrong terms by accident! By motion I meant the motion of the mass during oscillation, which in turn would displace the spring.

My intention is to stretch the springs (real springs) through half of their range of motion, allowing a +/- of the maximum displacement amplitude. That way the spring won't reach it's limit either way and will always be in tension.

What was confusing me is whether each springs inability to resist compressive displacement would alter the stiffness matrix... but I think Chester has cleared that up for me below

Thanks again for your help.
 
  • #6
Leo_m87 said:
Thanks for your reply! This was my intuitive sense of what would happen, but I managed to get confused anyway. Can you just confirm a little further...

If each spring was initially stretched by 10 mm (Apologies to anyone not using metric) there would be spring forces of +k(10) and -k(10) acting on each mass (equilibrium). Then, if the mass is displaced further to the right (+ve direction) by another 5 mm the forces now become +k(15) and -k(5)... Your point is that this has the same net effect as if the compressed spring pushed back ... in other words the other spring being in greater tension is producing the equivalent force?

And most importantly, this set-up won't affect the stiffness matrix that would be formed for the system pictured? Which, before this confusion, I was certain should be:
[2k, -k
-k, 2k]
Just so there is no question about it, let x1 and x2 represent the displacements of masses m1 and m2 relative to their initial equilibrium positions, respectfully. Please write down the force balances on m1 and m2.
 
  • #7
Chestermiller said:
Just so there is no question about it, let x1 and x2 represent the displacements of masses m1 and m2 relative to their initial equilibrium positions, respectfully. Please write down the force balances on m1 and m2.

To make my issue clearer I've take the example of a one mass, two spring system. (Attached image).

The first method in the image shows where my normal assumptions break down. The second method gets me the answer that I expect for this system, but when I apply this same approach of including x0 in a system with three springs I run into problems. The problem is the middle spring is extended by x0/2 on either side and I no longer seem to be getting suitable answers when I work through the algebra associated with that approach.

There's a mistake in my logic somewhere... any help would be great! Thanks
 

Attachments

  • Diagram.png
    Diagram.png
    15.2 KB · Views: 411
  • #8
I don't understand what you did, so I am going to do the force balances on the masses in the original figure myself. The initial tensions in the 3 springs are 10 k, and let ##x_1## represent the displacement of mass m1 to the right and let ##x_2## represent the displacement of mass 2 to the right. So, the force balance on mass 1 is:$$(10k+k(x_2-x_1))-(10k+kx_1)+F_1\sin\omega t=m\frac{d^2x_1}{dt^2}$$
The force balance on mass 2 is:$$(10k-kx_2)-(10k+k(x_2-x_1))+F_2\sin\omega t=m\frac{d^2x_2}{dt^2}$$
 
  • #9
Leo_m87 said:
There's a mistake in my logic somewhere... any help would be great! Thanks
Newton's law is ##F = m\ddot x## and not ##F = - m\ddot x##.

Wouldn't be good if the spring on the right exercised ##F = kx##. It should be ##F = -kx##.
And for the left spring you have ##-F = kx## -- matter of choosing and using your coordinate system consistently.
For one of the springs x is the extension, for the other it's the compression ! As you work out correctly in the second picture, where yo do write ##-kx##.
You make life difficult for yourself this way !
 
  • #10
Chestermiller said:
I don't understand what you did, so I am going to do the force balances on the masses in the original figure myself. The initial tensions in the 3 springs are 10 k, and let ##x_1## represent the displacement of mass m1 to the right and let ##x_2## represent the displacement of mass 2 to the right. So, the force balance on mass 1 is:$$(10k+k(x_2-x_1))-(10k+kx_1)+F_1\sin\omega t=m\frac{d^2x_1}{dt^2}$$
The force balance on mass 2 is:$$(10k-kx_2)-(10k+k(x_2-x_1))+F_2\sin\omega t=m\frac{d^2x_2}{dt^2}$$

Thanks Chester, I completely understand now. The initial tension that you assigned 10k is what I was referring to as Kx0, if that helps you understand what I did in the last picture.

In any event, I now realize my mistake in tackling the 2 mass system - I had incorrectly halved the initial tension for the middle spring and didn't realize why this was incorrect until going over your reply.

Thanks a lot for your help!
 

1. What is the definition of a spring in dynamics?

A spring is a mechanical component that is designed to store and release energy. It is typically made of a flexible material, such as metal or rubber, and can be stretched or compressed to store potential energy.

2. How do springs resist motion?

In dynamics, springs resist motion by exerting a force that is proportional to the displacement from its equilibrium position. This force is known as the spring force and it acts in the direction opposite to the displacement.

3. Do springs always resist motion?

No, springs do not always resist motion. It depends on the type of spring and the forces acting on it. For example, if a spring is stretched beyond its elastic limit, it can lose its ability to resist motion and may break or deform.

4. What happens to the energy stored in a spring when it is released?

When a spring is released, the energy stored in it is converted into kinetic energy, which results in motion. This is known as the law of conservation of energy, which states that energy cannot be created or destroyed, only transferred from one form to another.

5. Are there any real-life examples of springs resisting motion?

Yes, there are many real-life examples of springs resisting motion. Some common examples include trampolines, car suspensions, and pogo sticks. In each of these examples, the springs are designed to absorb and release energy to resist motion and provide a smoother experience.

Similar threads

Replies
5
Views
368
Replies
9
Views
2K
Replies
4
Views
2K
  • Introductory Physics Homework Help
Replies
4
Views
2K
  • Introductory Physics Homework Help
Replies
2
Views
2K
Replies
16
Views
1K
  • Advanced Physics Homework Help
Replies
3
Views
1K
Replies
5
Views
2K
  • Introductory Physics Homework Help
Replies
2
Views
3K
  • Introductory Physics Homework Help
Replies
19
Views
2K
Back
Top